LSAT and Law School Admissions Forum

Get expert LSAT preparation and law school admissions advice from PowerScore Test Preparation.

 Administrator
PowerScore Staff
  • PowerScore Staff
  • Posts: 8917
  • Joined: Feb 02, 2011
|
#59064
Please post your questions below!
 chian9010
  • Posts: 81
  • Joined: Jun 08, 2018
|
#60376
I have A and E in my contender. Could you explain why E is wrong?
 fersian
  • Posts: 19
  • Joined: Jan 19, 2019
|
#62019
I also would like to know why 'E' is wrong.
 Malila Robinson
PowerScore Staff
  • PowerScore Staff
  • Posts: 296
  • Joined: Feb 01, 2018
|
#62027
Hello chian9010, and fersian,
Answer E can be eliminated in the first few words. It is not necessarily true that we are trying to increase the number of salmon in the lake. What we know is that we want to (at least) protect the ones who are still there. So when Answer E begins by focusing on increasing the number of salmon it goes beyond what we need to strengthen this argument.
As an aside I think some of the confusion in E is that it seems likely that protecting the young and increasing the numbers should be a goal. But we technically don't know that we want more of the salmon. We only know that we want to try to preserve the numbers we have now.
Hope that helps,
Malila
 NotSureWhy
  • Posts: 8
  • Joined: Jun 08, 2019
|
#68161
Hi,
Thanks so much for that explanation. That makes sense. I was also skeptical of (E) because of the reference to prime reproductive stage, which was not mentioned in the stimulus. However, I saw something in (A) that I understood to avoid elsewhere and was a little confused. In answer choice (A), there is the phrase "preferable to any other method of protecting..." but it is not at all clear that it is preferable to any other method, only that it is preferable to trout fishing. I saw this as a red flag, but now wonder if this is a case of it just being a stronger answer? Your insight would be super helpful.

Thanks,
NotSureWhy
 Jeremy Press
PowerScore Staff
  • PowerScore Staff
  • Posts: 1000
  • Joined: Jun 12, 2017
|
#68184
Hi NotSureWhy,

This is a Strengthen-Principle question, so we're looking for a principle in the answer choices that will provide the strongest support for Lopez's conclusion. Since this is a Strengthen-Principle question, we're permitted to assume the validity of the principle, and we shouldn't worry overly much if the principle goes broader than the argument in the stimulus (so long as it gives Lopez a stronger basis for reaching the conclusion). In fact, theoretically, the stronger and broader the principle, the better, because it's more likely to "cover" the facts in the stimulus and thereby boost the conclusion. So the very thing you were suspicious about in answer choice A (the comparison to "any other method") is actually a very good thing in this question! It makes the shrimp method more definitively superior, since the trout method falls within the "any other method," given that it's a method focused on eliminating native species. The fact that the rule would also make the shrimp method superior to other kinds of techniques is not a problem, because the conclusion is still strengthened.

I hope this helps!

Jeremy
 NotSureWhy
  • Posts: 8
  • Joined: Jun 08, 2019
|
#68261
Got it got it. Thanks so much, Jeremy. This is much appreciated.

Very best,
NotSureWhy

Get the most out of your LSAT Prep Plus subscription.

Analyze and track your performance with our Testing and Analytics Package.